Notes from Lambert Herrgesell (zero815(AT)googlemail.com), May 13 2007 Proof of conjecture: Write sequence as a triangle, each row containing 2^k, k>= 0, elements. 1, 1, 2, 1, 4, 2, 4, 1, 8, 4, 8, 2, 12, 4, 8, 1, 18, 8, 16, 4, 26, 8, 16, 2, 34, 12, 24, 4, 36, 8, 16, .. Then the first entry each row is a(2^k-1), the last entry is a(2^(k+1)-2) So now each row starts with 1 and ends with 2^k. More generally, the second last term is 2^(k-1), the the forth last is 2^(k-2), and and so on. I.e. for 0 <= i < k, a(2^(k+1)-1 - 2^i) = 2^(k-i). (Which also explains the odd terms in a(n).) Rephrase the conjecture as: Each row of the triangle has a leading 1, all other term are even. In particular the last term of a row k>=0 is 2^k and for 0 <= i < k, a(2^(k+1)-1 - 2^i) = 2^(k-i). The leading 1 already follows from a(2^1-1)=1 and a(2^(k+1)-1) = a((2^(k+1)-2)/2) = a(2^k-1). The rest follows by another induction: The conjecture is obviously true for the first row. Now assume the conjecture correct up to row k-1. In row k, the odd indexes a(2^k-1), a(2^k+1), ... a(2^(k+1)-3) copy row k-1, since (2^k-2)/2 = 2^(k-1)-1 etc. and the conjecture holds by assumption. Note that the 2^(k-i) pattern is copied here. For the even indexed a(2^k), a(2^k+1), ... a(2^(k+1)-2) the following sums are considered: The last entry of the row a(2^(k+1)-2): sums a(2^k-1) and the a(2^k-1 - 2^i) pattern of row k-1 and since sum(i=0,k-1,2^i) = 2^k-1, a(2^(k+1)-2) = 2^k as desired. a(2^k) to a(2^(k+1)-2): The 1's are at indices 2^m-1 and 2^i is subtracted from the "start"-index, so for a(2^k+2j) to sum a 1, the sum (2^k+2j)/2 - 2^i = 2^m-1 has to be solved. (2^k+2j)/2 - 2^i = 2^m-1 <=> 2^(k-1)+j - 2^i = 2^m-1 <=> 2^(k-1)+j+1 = 2^m + 2^i Since 2^k - 2^i is even, there is no solution, if j is even. On the other hand, if j is odd and m <> i, then m and i are mutual exchangeable. I,e, if 2^(k-1)+j - 2^i = 2^m-1 then 2^(k-1)+j - 2^m = 2^i-1 and there are two 1's in the sum. In case m=i, j must be of the form 2^y-1. 2^(k-1)+2^y-1+1 = 2^(m+1) <=> 2^(k-1)+2^y = 2^(m+1) so k-1 = y = m. But 2^(k-1)+ 2^(k-1)-1 = 2^k-1 is not an index of row k-1. So a single one is not in the sum. And the conjecture holds.